University spokesperson: Most of the students surveyed at the university said they would prefer that the current food...

Sean on November 7, 2018

PT 80, S4, Q11

I don't understand why the answer is A, over C, or E. Could you please explain? Thank you!

Replies
Create a free account to read and take part in forum discussions.

Already have an account? log in

Mehran on November 7, 2018

Hi @smilde11, thanks for your post. Let's start with a careful assessment of the stimulus (as always). This one presents an argument; the conclusion is "we should rehire Hall Dining next year." Why? The premises given are that (1) most of the students surveyed at the university said they would prefer that the current food vendor be replaced (2) several vendors have publicly expressed interest in working for the university (3) the only alternative is Hall Dining (4) Hall Dining was the vendor up until this past year and (5) the preferences of the majority of students should be adhered to.

The question stem presents an Errors of Reasoning question. What is wrong with this argument, we are asked?

You can eliminate answer choice (C) because it is irrelevant. Notice that the argument in the stimulus does not say that the *only* factor to be considered is student preference. Rather, premise #3 is that Hall Dining is the only viable alternative to the current vendor. Thus, the author of this stimulus is not basing her conclusion solely on student preferences; rather, she is also considering what vendor(s) might actually be available to take the current vendor's place. Answer choice (C) can be eliminated because it is not an accurate description of the error in reasoning in this particular stimulus.

Answer choice (E) is logically identical to answer choice (C). Again: the author of the stimulus is *not* arguing that "a certain action" (rehiring Hall Dining) should be undertaken "merely on the grounds that it would be popular." Rather, the author of the stimulus explains, in the premise I have labeled #3, that only one vendor is a viable alternative to the current vendor. If answer choice (C) were correct, answer choice (E) would also be correct, and vice versa - and you can only have one right answer on any given LSAT question.

Answer choice (A) is correct because it points out a possible disconnect between the two argumentative premises supporting the ultimate conclusion: one about student preferences, and the other about which vendor is actually viable to replace the current vendor. Yes, we know that a majority of the students want the current vendor gone - but would the majority of students still feel that way if they knew that the only option is to revert back to Hill Dining? Maybe not, right? And this is the possibility that is overlooked by the argument. Put another way: the author of the stimulus never connects Premises 1 & 5, on the one hand, with Premises 2, 3 & 4, on the other. This is a weakness in her argument.

Hope this helps.

Carson on July 7, 2020

clearly not C; I ruled this answer choice out - along with the reasons stated above - because the passage literally says "for a variety of reasons" these other vendors won't work. However, my gripes with this problem is that in the first sentence the passage refers to "most students surveyed" and in the last sentence the passage refers to "the majority of students." I chose B because of this since it isn't clear that the sample is representative, instead, the sample is simply those who were surveyed and the argument refers to most of them. 'A' makes sense to me as well but I didn't choose it because I had to assume that the students knew there were more options available based on the "several vendors have publicly expressed..." statement and that they cared which vendor would take the place of the current vendor. As a general principle, most of the students surveyed are saying that if it's the current vendor, then it should be replaced. The inference is: it is the current vendor (CV), therefore it should be replaced (RCV). I cannot infer from this that the replacing vendor has to be a particular type of vendor or have certain attributes and this is ultimately what the answer A does. What's more, the preference of the students and the information provided by the author seem to be valid. For example, CV ---> RCV ("the only vendor," i.e. if it's the RCV, then it must be...) ---> Hall Dining Services. So, yeah, I thought this was a five star question for flawed questions and I must be interpreting something wrong... which I would love to know where I'm going wrong because I've unfortunately made an argument that seems only to support B.

Carson on July 7, 2020

I mean, answer-A 100% is a flaw, I just don't see how the argument is more vulnerable to A than B given the structure of this question.

Carson on July 7, 2020

I am interpreting the 'until' condition for Hall Dining Services to mean that they are not the current vendor.